6
$\begingroup$

I'm studying Richter's "A new elementary proof of the Prime Number Theorem" paper, and I'm finding some problems understanding some parts of it. For example, I don't see how to get, in Lemma 3.4, that we can find two non-consecutive $$a,b\in\{0,\ldots,K-1\}$$ such that the intervals $$(8^{t+aε^4} , 8^{t+(a+1)ε^4} ], (8^{t+bε^4} , 8^{t+(b+1)ε^4}]$$ contain at least $$O\left(\frac{ε^48^n}{n}\right)$$ many primes.

Here's the reference to the paper: https://arxiv.org/abs/2002.03255. Can someone help me?

$\endgroup$
1
  • 3
    $\begingroup$ What bound on the error term does his proof give? $\endgroup$ Commented Dec 10, 2024 at 17:53

1 Answer 1

16
$\begingroup$

The proof of Lemma 3.4 uses the constant $K=[\epsilon^{-3}]$ which should be your clue. After the definition of $K$ the author covers $(8^t,8^{t+\epsilon})$ (which contains at least $\epsilon 8^n/(2n)$ primes by the choice of $t$ as per previous paragraph and Lemma $3.1$) with $K$ intervals each having at most $O(\epsilon^2 8^n/(n))$ primes by Lemma $3.3$ (see the at most vs at least above)

In particular, if only one (or none) of those intervals contains at least $O(\epsilon^4 8^n/(n))$ primes (so the rest contain at most $o(\epsilon^4 8^n/(n))$ primes) and since there $K \approx \epsilon^{-3}$ such intervals, the total number of primes in $(8^t,8^{t+\epsilon})$ can be at most $$O(\epsilon^2 8^n/n)+\epsilon^{-3}o(\epsilon^4 8^n/n)=o(\epsilon 8^n/(2n))$$ and that is a contradiction with the above. So the claim follows - and actually one gets that at least of the order $\epsilon^{-1}=K^{1/3}$ such nonconsecutive $a,b,....$ exist

Edit to answer the question from comment: What I mean by $O(\epsilon^2 8^n/n)+\epsilon^{-3}o(\epsilon^4 8^n/n)=o(\epsilon 8^n/(2n))$ is that in the paper Lemma $3.3$ bounds the number of primes in any one of those smaller intervals by $C\epsilon^2 8^n/n$ where $C=C_{\epsilon_0}$ is valid for all $\epsilon <\epsilon_0$ and all $x$ (hence all $n,t$ intervals) large enough independent of $\epsilon_0$; so now if for a positive constant to be chosen later $D$, there is only at most one interval of those $K=[\epsilon^{-3}]$ that has at least $D\epsilon^4 8^n/(n)$ primes, then the total number of primes in the large interval is at most $C\epsilon^2 8^n/n+DK\epsilon^4 8^n/n \le D_1\epsilon 8^n/(2n)$ for a given $D_1$ depending only on $D, \epsilon_0$ and going to zero when $D \to 0$ and $\epsilon \to 0$ independently; since we know that number to be at least $C_1\epsilon 8^n/(2n)$, choosing an $\epsilon_1$ small enough to have the $C\epsilon^2 8^n/ <(C_1/2)\epsilon 8^n/(2n), \epsilon<\epsilon_1$ and $D$ small enough so $DK\epsilon^4 8^n/n<(C_1/2)\epsilon 8^n/(2n)$ (here the maybe a bit imprecise $o(\epsilon^4 8^n/(n))$ is used) so $D_1 <C_1$ gives a contradiction.

Note that this argument immediately shows that the number of intervals with "many" primes (at least $D\epsilon^4 8^n/(n)$ for a small enough but fixed $D>0$ and all $\epsilon<\epsilon_1$) not only has to be at least $2$ but at least some $cK^{1/3}$, since if their number is at most $M$ then we need for example $4MC\epsilon \ge C_1$ since otherwise we can make the second term $DK\epsilon^4 8^n/n$ small enough to get a contradiction and this gives us $M \ge c_0/\epsilon \ge cK^{1/3}$

$\endgroup$
5
  • 1
    $\begingroup$ i understand your answer but still I'm not 100% sure about this identity $$O(\epsilon^2 8^n/n)+\epsilon^{-3}o(\epsilon^4 8^n/n)=o(\epsilon 8^n/(2n)).$$ If it doesn't bother you could explain to me how did you get the RHS from the LHS?. @conrad $\endgroup$ Commented Dec 12, 2024 at 11:12
  • $\begingroup$ Since you are already familiar with the paper, if you don't mind giving me an hand with another part of it, I have some doubt about the proof of proposition 3.1. In particular, when he estimates the number of primes in $$(\frac{8^x}{2^{n+1}},\frac{8^x}{2^n})$$. I'm not sure if the estimations is right to begin with, cause the application of the lemma itself would give a different bound $\endgroup$ Commented Dec 16, 2024 at 16:51
  • $\begingroup$ thank you in advance $\endgroup$ Commented Dec 16, 2024 at 17:46
  • 1
    $\begingroup$ put the answer for that $\endgroup$ Commented Dec 16, 2024 at 18:18
  • $\begingroup$ If you are interested in the answer to the question In the comments you can check it up there: mathoverflow.net/questions/484247/…. Thanks again to @conrad for the help $\endgroup$ Commented Dec 16, 2024 at 18:21

You must log in to answer this question.

Start asking to get answers

Find the answer to your question by asking.

Ask question

Explore related questions

See similar questions with these tags.